Mathcenter Forum  

Go Back   Mathcenter Forum > คณิตศาสตร์โอลิมปิก และอุดมศึกษา > อสมการ
สมัครสมาชิก คู่มือการใช้ รายชื่อสมาชิก ปฏิทิน ข้อความวันนี้

ตั้งหัวข้อใหม่ Reply
 
เครื่องมือของหัวข้อ ค้นหาในหัวข้อนี้
  #61  
Old 04 มีนาคม 2007, 21:33
devilzoa devilzoa ไม่อยู่ในระบบ
จอมยุทธ์หน้าใหม่
 
วันที่สมัครสมาชิก: 10 ธันวาคม 2005
ข้อความ: 83
devilzoa is on a distinguished road
Post

28.สมมติให้ $a=\frac{x}{y},b=\frac{y}{z},c=\frac{z}{x} , a,b,c>0$
ใช้ Am-Gm จะได้ $a+b+c\ge3$ จะได้ว่า $\frac{(a+b+c)^{2}}{3}\ge a+b+c$
ใช้ power mean ${a^{2}+b^{2}+c^{2}}\ge\frac{(a+b+c)^{2}}{3}\ge a+b+c$

**มีสั้นกว่านี้ไหมครับ ชี้แนะด้วยครับ ผมยังมือใหม่อยู่ \_/?

04 มีนาคม 2007 21:34 : ข้อความนี้ถูกแก้ไขแล้ว 1 ครั้ง, ครั้งล่าสุดโดยคุณ devilzoa
ตอบพร้อมอ้างอิงข้อความนี้
  #62  
Old 05 มีนาคม 2007, 00:54
nooonuii nooonuii ไม่อยู่ในระบบ
ผู้พิทักษ์กฎทั่วไป
 
วันที่สมัครสมาชิก: 25 พฤษภาคม 2001
ข้อความ: 6,408
nooonuii is on a distinguished road
Post

อ้างอิง:
ข้อความเดิมของคุณ devilzoa:
28.สมมติให้ $a=\frac{x}{y},b=\frac{y}{z},c=\frac{z}{x} , a,b,c>0$
ใช้ Am-Gm จะได้ $a+b+c\ge3$ จะได้ว่า $\frac{(a+b+c)^{2}}{3}\ge a+b+c$
ใช้ power mean ${a^{2}+b^{2}+c^{2}}\ge\frac{(a+b+c)^{2}}{3}\ge a+b+c$

**มีสั้นกว่านี้ไหมครับ ชี้แนะด้วยครับ ผมยังมือใหม่อยู่ \_/?

ไม่รู้ว่าสั้นรึเปล่านะครับ แต่ผมมีอีกสองวิธี

First Solution :
โดย AM-GM จะได้ $a^2+b^2+c^2\geq 3$
โดยอสมการโคชีจะได้ $a+b+c\leq\sqrt{3(a^2+b^2+c^2)}\leq a^2+b^2+c^2$

Second Solution :
$$(a^2-a)+(b^2-b)+(c^2-c)=(a-1)^2+(b-1)^2+(c-1)^2+(a+b+c-3)\geq 0$$
__________________
site:mathcenter.net คำค้น
ตอบพร้อมอ้างอิงข้อความนี้
  #63  
Old 05 มีนาคม 2007, 08:52
nooonuii nooonuii ไม่อยู่ในระบบ
ผู้พิทักษ์กฎทั่วไป
 
วันที่สมัครสมาชิก: 25 พฤษภาคม 2001
ข้อความ: 6,408
nooonuii is on a distinguished road
Post

มาเติมโจทย์ครับ

31. $a,b,c>0$
$$\frac{a}{b+c}+\frac{b}{c+a}+\frac{c}{a+b}\geq\frac{3}{2}$$

32. $a,b,c,d>0$

$$\frac{a}{b+c}+\frac{b}{c+d}+\frac{c}{d+a}+\frac{d}{a+b}\geq 2$$

33. $a,b,c,d\geq 0$
$$\sqrt{(ad+bc)(ab+cd)}\leq \frac{(a+b+c+d)^2}{8}$$

34. $a,b,c\geq 0$
$$a^4+b^4+c^4\geq abc(a+b+c)$$

35. $a,b,c>0$
$$a^bb^cc^a\leq \Big(\frac{a+b+c}{3}\Big)^{a+b+c}\leq a^ab^bc^c$$
__________________
site:mathcenter.net คำค้น
ตอบพร้อมอ้างอิงข้อความนี้
  #64  
Old 05 มีนาคม 2007, 15:50
devilzoa devilzoa ไม่อยู่ในระบบ
จอมยุทธ์หน้าใหม่
 
วันที่สมัครสมาชิก: 10 ธันวาคม 2005
ข้อความ: 83
devilzoa is on a distinguished road
Post

ข้อ 31 Chebyshev's inequality และ Am-Hm
$3(\frac{a}{b+c}+\frac{b}{a+c}+\frac{c}{a+b})\ge (a+b+c)(\frac{1}{b+c}+\frac{1}{a+c}+\frac{1}{a+b})\ge\frac{9}{2}$
จะได้ $\frac{a}{b+c}+\frac{b}{a+c}+\frac{c}{a+b}\ge\frac{3}{2}$

ข้อ 32 น่าจะทำแบบเดียวกันกับข้อ 31
ข้อ 33 ใช้ Am-Gm $\frac{(ad+bc)+(ab+cd)}{2}=\frac{(a+c)(b+d)}{2}\ge\sqrt{(ad+bc)(ab+cd)}$
และ $\frac{(a+b+c+d)^{2}}{4}\ge(a+c)(b+d)$
ดังนั้น $\frac{(a+b+c+d)^{2}}{8}\ge\frac{(a+c)(b+d)}{2}\ge\sqrt{(ad+bc)(ab+cd)}$

ข้อ 34 Am-Gm $(\frac{a+b+c}{3})^{3}\ge abc$ ดังนั้น $\frac{(a+b+c)^4}{(3)^{3}}\ge abc(a+b+c)$
Power mean $a^{4}+b^{4}+c^{4}\ge\frac{(a+b+c)^4}{(3)^{3}}\ge abc(a+b+c)$

05 มีนาคม 2007 18:13 : ข้อความนี้ถูกแก้ไขแล้ว 3 ครั้ง, ครั้งล่าสุดโดยคุณ devilzoa
ตอบพร้อมอ้างอิงข้อความนี้
  #65  
Old 06 มีนาคม 2007, 09:34
nooonuii nooonuii ไม่อยู่ในระบบ
ผู้พิทักษ์กฎทั่วไป
 
วันที่สมัครสมาชิก: 25 พฤษภาคม 2001
ข้อความ: 6,408
nooonuii is on a distinguished road
Post

31. Alternative Solution:
ใช้อสมการโคชีจะได้
$$ab+bc+ca\leq a^2+b^2+c^2 \Rightarrow 3(ab+bc+ca)\leq (a+b+c)^2$$

ใช้อสมการโคชีอีกครั้งจะได้ว่า

$\begin{array}{rcl} a+b+c & = & \displaystyle{ \sqrt{\frac{a}{b+c}}\cdot \sqrt{a(b+c)}+\sqrt{\frac{b}{c+a}}\cdot \sqrt{b(c+a)}+\sqrt{\frac{c}{a+b}}\cdot \sqrt{c(a+b)} } \\ & \leq & \displaystyle{ \sqrt{\frac{a}{b+c} + \frac{b}{c+a}+\frac{c}{a+b}} \sqrt{2(ab+bc+ca)}} \\
& \leq & \displaystyle{ \sqrt{\frac{a}{b+c} + \frac{b}{c+a}+\frac{c}{a+b}} \sqrt{\frac{2}{3}(a+b+c)^2} } \\
& = & \displaystyle{ \sqrt{\frac{2}{3}\Big(\frac{a}{b+c} + \frac{b}{c+a}+\frac{c}{a+b}\Big)} \Big(a+b+c \Big) } \end{array}$

จัดรูปจะได้อสมการตามต้องการ

32. ไม่น่าจะใช้ Chebychev's inequality ได้ครับ เพราะจะมีปัญหาตอนเรียงค่า

34. Alternative Solution:
ใช้อสมการโคชีสองครั้ง
$\begin{array}{rcl} abc(a+b+c) & = & ab\cdot ca + bc\cdot ab + ca\cdot bc \\ & \leq & a^2b^2+b^2c^2+c^2a^2 \\
& \leq & a^4+b^4+c^4 \end{array}$
__________________
site:mathcenter.net คำค้น
ตอบพร้อมอ้างอิงข้อความนี้
  #66  
Old 05 พฤษภาคม 2007, 06:03
Punk Punk ไม่อยู่ในระบบ
ลมปราณบริสุทธิ์
 
วันที่สมัครสมาชิก: 10 เมษายน 2005
ข้อความ: 108
Punk is on a distinguished road
Default

ไม่ได้จะมาทำโจทย์แต่มาเพิ่มโจทย์ครับ ข้อนี้ไม่ยากเผอิญไปได้แนวคิดมาจากการพิสูจน์ isodiametric inequality ครับ

36. ให้ $E$ เป็นเซตของจุดบนระนาบ ซึ่งสมมาตรกับจุดกำเนิด และระยะระหว่างสองจุดใดๆไม่เกิน $d$ จงพิสูจน์ว่าพื้นที่ของเซต $E$ สอดคล้องอสมการ
\[
Area(E)\leq\pi\frac{d^2}{4}
\]

05 พฤษภาคม 2007 06:04 : ข้อความนี้ถูกแก้ไขแล้ว 2 ครั้ง, ครั้งล่าสุดโดยคุณ Punk
ตอบพร้อมอ้างอิงข้อความนี้
  #67  
Old 05 พฤษภาคม 2007, 07:12
warut warut ไม่อยู่ในระบบ
กระบี่ไร้สภาพ
 
วันที่สมัครสมาชิก: 24 พฤศจิกายน 2001
ข้อความ: 1,627
warut is on a distinguished road
Default

ผมว่า ข้อ 36. เป็นโจทย์ที่ง่ายที่สุดที่คุณ Punk เคยเอามาโพสต์เลยนะครับ (ทุกทีแค่อ่านโจทย์อย่างเดียว ก็ยังยากที่จะเข้าใจเลย)

ให้ $(x,y)\in E$ ดังนั้น $(-x,-y)\in E$ ด้วย เนื่องจาก $E$ มีสมมาตรกับจุดกำเนิด

ให้ $x^2+y^2=r^2$ โดยที่ $r\ge0$ เราจะได้ระยะทางระหว่าง $(x,y)$ กับ $(-x,-y)$ คือ $2r$ ดังนั้น $2r\le d$

แสดงว่า $x^2+y^2\le \left( \frac{d}{2} \right)^2$ เราจึงได้ว่า $E\subseteq \{ (x,y)\in \mathbb R^2 \mid x^2+y^2\le \left( \frac{d}{2} \right)^2 \}$ และ $$Area(E)\le\pi\frac{d^2}{4}$$
ตอบพร้อมอ้างอิงข้อความนี้
  #68  
Old 05 พฤษภาคม 2007, 08:36
Punk Punk ไม่อยู่ในระบบ
ลมปราณบริสุทธิ์
 
วันที่สมัครสมาชิก: 10 เมษายน 2005
ข้อความ: 108
Punk is on a distinguished road
Default

แฮะๆๆๆคิดอยู่แล้วเชียวว่าต้องเรียบร้อย(โรงเรียน)คุณ warut ข้อนี้ผมว่ามัน tricky นิดหน่อยนะครับ แบบที่เค้าว่ากันว่าคณิตศาสตร์บางครั้งต้องตีลังกาคิด
ตอบพร้อมอ้างอิงข้อความนี้
  #69  
Old 05 พฤษภาคม 2007, 09:01
nooonuii nooonuii ไม่อยู่ในระบบ
ผู้พิทักษ์กฎทั่วไป
 
วันที่สมัครสมาชิก: 25 พฤษภาคม 2001
ข้อความ: 6,408
nooonuii is on a distinguished road
Default

ในที่สุดความลับสวรรค์ก็เปิดเผย มิน่าผมคิดเท่าไรก็คิดไม่ออก เราต้องตีลังกาคิดนี่เองครับ

ป.ล. พี่ Punk ไม่ส่งรูปมาโชว์บ้างเลยครับ จะได้เก็บไว้เป็นที่ระลึก
__________________
site:mathcenter.net คำค้น
ตอบพร้อมอ้างอิงข้อความนี้
  #70  
Old 19 พฤษภาคม 2007, 09:07
kanakon's Avatar
kanakon kanakon ไม่อยู่ในระบบ
บัณฑิตฟ้า
 
วันที่สมัครสมาชิก: 31 ตุลาคม 2006
ข้อความ: 523
kanakon is on a distinguished road
Default

เห็นกระทู้เงียบๆ เลยเอามาฝากครับ

37. If a,b,c,...,k are +ve quantities. Prove that

$$\left(\frac{a+b+c+...+k}{n} \,\right)^{a+b+c+...+k}<{a^a}{b^b}{c^c}...{k^k}$$
__________________
ค ว า ม รั บ ผิ ด ช อ บ

$$|I-U|\rightarrow \infty $$
ตอบพร้อมอ้างอิงข้อความนี้
  #71  
Old 19 พฤษภาคม 2007, 09:49
nooonuii nooonuii ไม่อยู่ในระบบ
ผู้พิทักษ์กฎทั่วไป
 
วันที่สมัครสมาชิก: 25 พฤษภาคม 2001
ข้อความ: 6,408
nooonuii is on a distinguished road
Default

อ้างอิง:
ข้อความเดิมเขียนโดยคุณ kanakon View Post
เห็นกระทู้เงียบๆ เลยเอามาฝากครับ

37. If $a_1,a_2,...,a_n$ are positive quantities. Prove that

$$\left(\frac{a_1+a_2+\cdots+a_n}{n} \,\right)^{a_1+a_2+\cdots+a_n} \leq {a_1^{a_1}}{a_2^{a_2}}\cdots {a_n^{a_n}}$$
__________________
site:mathcenter.net คำค้น
ตอบพร้อมอ้างอิงข้อความนี้
  #72  
Old 01 สิงหาคม 2007, 22:37
kanakon's Avatar
kanakon kanakon ไม่อยู่ในระบบ
บัณฑิตฟ้า
 
วันที่สมัครสมาชิก: 31 ตุลาคม 2006
ข้อความ: 523
kanakon is on a distinguished road
Default

ก่อนสอบรอบสองผมขอทิ้งโจทย์ประดับบอร์ดอีกข้อนะครับ
เพราะบอร์ด อสมการ เงียบเชียบเหลือเกินไม่ค่อยมีคนสนใจเลย

Let $n\geq 3$ be an integer. Let $t_1,t_2,t_3,...t_n$ be posiive real integer such that
$$(n^2+1)>(t_1+t_2+...+t_n)(\frac{1}{t_1}+\frac{1}{t_2}+...+\frac{1}{t_n} )$$
Show that $t_i,t_j,t_k$ are side lenghts of a triangle for all $i,j,k$ with $1\leq i<j<k\leq n$ [IMO 2004]
__________________
ค ว า ม รั บ ผิ ด ช อ บ

$$|I-U|\rightarrow \infty $$
ตอบพร้อมอ้างอิงข้อความนี้
  #73  
Old 27 สิงหาคม 2007, 00:32
Spotanus's Avatar
Spotanus Spotanus ไม่อยู่ในระบบ
จอมยุทธ์หน้าหยก
 
วันที่สมัครสมาชิก: 28 มีนาคม 2007
ข้อความ: 171
Spotanus is on a distinguished road
Default

เพิ่งคิดโจทย์ตะกี้เอง พอดีเห็นคุณ nooonuii บอกให้ลองมาเล่นดูครับ
37.อสมการ
ตอบพร้อมอ้างอิงข้อความนี้
  #74  
Old 29 สิงหาคม 2007, 20:50
tatari/nightmare's Avatar
tatari/nightmare tatari/nightmare ไม่อยู่ในระบบ
ลมปราณคุ้มครองร่าง
 
วันที่สมัครสมาชิก: 29 กรกฎาคม 2007
ข้อความ: 276
tatari/nightmare is on a distinguished road
Default

อ้างอิง:
ข้อความเดิมเขียนโดยคุณ kanakon View Post
Let $n\geq 3$ be an integer. Let $t_1,t_2,t_3,...t_n$ be posiive real integer such that
$$(n^2+1)>(t_1+t_2+...+t_n)(\frac{1}{t_1}+\frac{1}{t_2}+...+\frac{1}{t_n} )$$
Show that $t_i,t_j,t_k$ are side lenghts of a triangle for all $i,j,k$ with $1\leq i<j<k\leq n$ [IMO 2004]
$(t_1+t_2+...+t_n)(\frac{1}{t_1}+\frac{1}{t_2}+...+\frac{1}{t_n})=n+\sum_{i<j}^{}(\frac{t_i}{t_j}+\frac{t_j}{t_i})
=n+t_1(\frac{1}{t_2}+\frac{1}{t_3})+\frac{1}{t_1}(t_2+t_3)+\sum_{i<j,(i,j)\not\in{(1,2),(1,3)}}^{}(\frac{t_i}{t_j}+\frac{t_j}{t_ i})$
จาก $$\frac{1}{t_2}+\frac{1}{t_3}\geq\frac{2}{\sqrt{t_{2}t_{3}}}$$
$$t_2+t_3\geq 2\sqrt{t_{2}t_{3}}$$ ( AM-GM)
และ $\frac{t_i}{t_j}+\frac{t_j}{t_i}\geq2$ ให้ $a=\frac{t_1}{\sqrt{t_{2}t_{3}}}$ ได้ $a>0$ จากโจทย์ได้
$n^2+1>(t_1+t_2+...+t_n)(\frac{1}{t_1}+\frac{1}{t_2}+...+\frac{1}{t_n})\geq n+\frac{2t_1}{\sqrt{t_{2}t_{3}}}+\frac{2\sqrt{t_{2}t_{3}}}{t_1}+2[\binom{n}{2}-2]=2a+\frac{2}{a}+n^2-4$
$\therefore 2a+\frac{2}{a}-5< 0$ นั่นคือ $2a^2-5a+2<0$ จะได้ $\frac{1}{2}<a<2$
นั่นคือ $\frac{t_1}{\sqrt{t_{2}t_{3}}}=a<2$
$\therefore t_1<2\sqrt{t_{2}t_{3}}$ แต่จาก $2\sqrt{t_{2}t_{3}}\leq t_2+t_3$
ฉะนั้น $t_1<t_2+t_3$
__________________
AL-QAEDA(เอXข้างหน้า!!)!!!!!!!!!!
ถึง บิน ลาเดนจะลาโลกไปแล้ว แต่เรายังมีผู้นำ jihad คนใหม่....อย่าง
อับดุล อาบาเร่ คราลิดทากัน...เราจะใช้รถดูดส้XXเป็นคาร์บอม!!!จงพลีชีพเพื่อผู้นำของเรา!!!!!!!

BOOM!!!!!!!!!!!!!!!!!!!!!

29 สิงหาคม 2007 20:51 : ข้อความนี้ถูกแก้ไขแล้ว 1 ครั้ง, ครั้งล่าสุดโดยคุณ tatari/nightmare
ตอบพร้อมอ้างอิงข้อความนี้
  #75  
Old 29 สิงหาคม 2007, 20:57
tatari/nightmare's Avatar
tatari/nightmare tatari/nightmare ไม่อยู่ในระบบ
ลมปราณคุ้มครองร่าง
 
วันที่สมัครสมาชิก: 29 กรกฎาคม 2007
ข้อความ: 276
tatari/nightmare is on a distinguished road
Default

39.Let x,y,zbe positive real numbers such that $xyz\geq 1$,Prove that $$\frac{x^5-x^2}{x^5+y^2+z^2}+\frac{y^5-y^2}{y^5+z^2+x^2}+\frac{z^5-z^2}{z^5+x^2+y^2}\geq 0$$
__________________
AL-QAEDA(เอXข้างหน้า!!)!!!!!!!!!!
ถึง บิน ลาเดนจะลาโลกไปแล้ว แต่เรายังมีผู้นำ jihad คนใหม่....อย่าง
อับดุล อาบาเร่ คราลิดทากัน...เราจะใช้รถดูดส้XXเป็นคาร์บอม!!!จงพลีชีพเพื่อผู้นำของเรา!!!!!!!

BOOM!!!!!!!!!!!!!!!!!!!!!
ตอบพร้อมอ้างอิงข้อความนี้
ตั้งหัวข้อใหม่ Reply


หัวข้อคล้ายคลึงกัน
หัวข้อ ผู้ตั้งหัวข้อ ห้อง คำตอบ ข้อความล่าสุด
Algebra Marathon nooonuii พีชคณิต 199 20 กุมภาพันธ์ 2015 10:08
Trigonometric Marathon Mastermander พีชคณิต 251 24 พฤศจิกายน 2013 21:21
Calculus Marathon (2) nongtum Calculus and Analysis 134 03 ตุลาคม 2013 16:32
Marathon Mastermander ฟรีสไตล์ 6 02 มีนาคม 2011 23:19
Calculus Marathon nooonuii Calculus and Analysis 222 26 เมษายน 2008 03:52


กฎการส่งข้อความ
คุณ ไม่สามารถ ตั้งหัวข้อใหม่ได้
คุณ ไม่สามารถ ตอบหัวข้อได้
คุณ ไม่สามารถ แนบไฟล์และเอกสารได้
คุณ ไม่สามารถ แก้ไขข้อความของคุณเองได้

vB code is On
Smilies are On
[IMG] code is On
HTML code is Off
ทางลัดสู่ห้อง


เวลาที่แสดงทั้งหมด เป็นเวลาที่ประเทศไทย (GMT +7) ขณะนี้เป็นเวลา 23:08


Powered by vBulletin® Copyright ©2000 - 2024, Jelsoft Enterprises Ltd.
Modified by Jetsada Karnpracha